Xem bài viết đơn
Old 04-07-2012, 10:45 PM   #762
Hoang Nguyen
+Thành Viên+
 
Tham gia ngày: Jun 2011
Bài gởi: 77
Thanks: 25
Thanked 6 Times in 5 Posts
1.Cho n lớn hơn hoặc bằng 2.Giả sử $a_1,a_2...,a_n $ và $b_1,b_2...,b_n $ là 2n số không âm sao cho $\sum^{n}_{i=1}a_i=\sum^{n}_{i=1}b_i=1. $ Đồng thời $b_i\le\frac{n-1}{n} $.CMR
$b_1a_1a_2...a_n+...+a_1a_2...a_{k-1}b_{k}a_{k+1}...a_n+a_1a_2...a_{n-1}b_n \le \frac{1}{n(n-1)^{n-2}} $
2.Giả sử $a_1,a_2...,a_n $ và $b_1,b_2...,b_n $ là 2n số
thưc nằm trong khoảng [1001;2002].Giả sử $\sum^{n}_{i=1}a_i^2=\sum^{n}_{i=1}b_i^2=1. $
CMR:$\sum^{n}_{i=1}\frac{a_i^3}{b_i}\le\frac{17}{10} \sum^{n}_{i=1}a_i^2 $
3.Cho trước n.Tìm m để
$\sum^{n}_{k=1}\frac{m^2+4km}{a_1+a_2+...+a_k}\le 2500\sum^{n}_{k=1}(\frac{1}{a_i}) $
[RIGHT][I][B]Nguồn: MathScope.ORG[/B][/I][/RIGHT]
 
Hoang Nguyen is offline  
 
[page compression: 7.39 k/8.36 k (11.57%)]